Difference between revisions of "2013 AMC 12B Problems/Problem 22"

(Blanked the page)
(Tag: Blanking)
Line 1: Line 1:
==Problem==
 
Let <math>m>1</math> and <math>n>1</math> be integers. Suppose that the product of the solutions for <math>x</math> of the equation
 
<cmath> 8(\log_n x)(\log_m x)-7\log_n x-6 \log_m x-2013 = 0 </cmath>
 
is the smallest possible integer. What is <math>m+n</math>?
 
  
<math> \textbf{(A)}\ 12\qquad\textbf{(B)}\ 20\qquad\textbf{(C)}\ 24\qquad\textbf{(D)}\ 48\qquad\textbf{(E)}\ 272 </math>
 
 
==Solution==
 
Rearranging logs, the original equation becomes
 
<cmath>\frac{8}{\log n \log m}(\log x)^2 - \left(\frac{7}{\log n}+\frac{6}{\log m}\right)\log x - 2013 = 0</cmath>
 
 
By Vieta's Theorem, the sum of the possible values of <math>\log x</math> is <math>\frac{\frac{7}{\log n}+\frac{6}{\log m}}{\frac{8}{\log n \log m}} = \frac{7\log m + 6 \log n}{8} = \log \sqrt[8]{m^7n^6}</math>. But the sum of the possible values of <math>\log x</math> is the logarithm of the product of the possible values of <math>x</math>. Thus the product of the possible values of <math>x</math> is equal to <math>\sqrt[8]{m^7n^6}</math>.
 
 
It remains to minimize the integer value of <math>\sqrt[8]{m^7n^6}</math>. Since <math>m, n>1</math>, we can check that <math>m = 2^2</math> and <math>n = 2^3</math> work. Thus the answer is <math>4+8 = \boxed{\textbf{(A)}\ 12}</math>.
 
 
==Video Solution==
 
For those who prefer a video solution: https://www.youtube.com/watch?v=vX0y9lRv9OM&t=312s
 
 
== See also ==
 
{{AMC12 box|year=2013|ab=B|num-b=21|num-a=23}}
 
{{MAA Notice}}
 

Revision as of 23:42, 6 December 2021